LSAT and Law School Admissions Forum

Get expert LSAT preparation and law school admissions advice from PowerScore Test Preparation.

 Administrator
PowerScore Staff
  • PowerScore Staff
  • Posts: 8916
  • Joined: Feb 02, 2011
|
#73471
Complete Question Explanation
(The complete setup for this game can be found here: https://forum.powerscore.com/lsat/viewtopic.php?t=3901)

The correct answer choice is (D)

If Q performs at 12, then P must perform at 6. In addition, from the actions of the last rule, a folk group must play at 10, and from the third rule we can then deduce that two folk groups play at 10, leading to the following setup:
D05_Game_#4_#21_diagram_1.png
G, H, L, and T are the remaining groups that must still perform. From the second rule, G must perform earlier than H, and so G performs at 8 and H performs at 10. In order to conform to the third rule, L, the only other folk group, must then also perform at 10, and T is left to perform at 8. These inferences lead to the following setup:
D05_Game_#4_#21_diagram_2.png
Thus, although we cannot determine the stages that G, H, L, and T perform on, we can determine the times that they perform.

Answer choice (A) is incorrect because P must perform first on the north stage.

Answer choice (B) is incorrect because H cannot perform at 8, and T cannot perform at 10.

Answer choice (C) is incorrect because G cannot perform at 10.

Answer choice (E) is incorrect because from the fourth rule L and T cannot perform on the same stage.

Answer choice (D) is thus proven correct.
You do not have the required permissions to view the files attached to this post.

Get the most out of your LSAT Prep Plus subscription.

Analyze and track your performance with our Testing and Analytics Package.